Đến nội dung

hoctrocuaHolmes nội dung

Có 1000 mục bởi hoctrocuaHolmes (Tìm giới hạn từ 16-05-2020)



Sắp theo                Sắp xếp  

#578400 Ảnh thành viên

Đã gửi bởi hoctrocuaHolmes on 04-08-2015 - 13:43 trong Góc giao lưu

Đà >:) 

''Phét en pho ri ớt''  :mellow:  :mellow:




#578403 Ảnh thành viên

Đã gửi bởi hoctrocuaHolmes on 04-08-2015 - 13:50 trong Góc giao lưu

BỮA EM IB FB CHỊ CHỊ CÒN KH REP EM, SAO EM DÁM KHÔNG PHÉT ĐƯỢC -_- 

Face dạo này chẳng muốn lên nữa bạn à,đang trong quá trình ''đóng băng''  :luoi: Với lại quan trọng là bố mẹ không cho dùng  >:)




#577255 Ảnh thành viên

Đã gửi bởi hoctrocuaHolmes on 31-07-2015 - 22:17 trong Góc giao lưu

Mà cái ảnh báo họ chụp được 1 tấm là up luôn chứ chẳng chọn gì cả,đúng cái lúc em nhìn đâu đâu,mặt còn ... :(  nữa




#578395 Ảnh thành viên

Đã gửi bởi hoctrocuaHolmes on 04-08-2015 - 13:39 trong Góc giao lưu

Thế bạn Khoa có muốn tung hình đáng yêu của bạn không ạ? :)) 

Tung đi ai cấm  :D

Spoiler




#577962 Ảnh thành viên

Đã gửi bởi hoctrocuaHolmes on 02-08-2015 - 21:26 trong Góc giao lưu

EM CŨNG CÓ ẢNH -_- NHÌN ĐI NHÌN LẠI CÓ DUY NHẤT TẤM NÀY GIỐNG NỮ NHI NHẤT -_- 

 

 

11229407_1584847501804553_70227373409312

Lớp mấy rồi nloan2k1  :icon6:




#577256 Ảnh thành viên

Đã gửi bởi hoctrocuaHolmes on 31-07-2015 - 22:18 trong Góc giao lưu

Sướng thế đã được đi máy bay rùi anh còn chưa nhìn thấy cái sân bay bh

Một năm 1 lần thôi,mà đi máy bay chán chết đi được,ưu điểm là cô tiếp viên xinh thôi  :D




#577263 Ảnh thành viên

Đã gửi bởi hoctrocuaHolmes on 31-07-2015 - 22:26 trong Góc giao lưu

Sao đi nhiều vây, anh đến bây giờ mới chỉ được đi có 1 lần  :wacko:

Đi vì nghĩa vụ thôi anh,cũng ko sung sướng lắm đâu  :(




#577279 Ảnh thành viên

Đã gửi bởi hoctrocuaHolmes on 31-07-2015 - 22:33 trong Góc giao lưu

đi máy bay ù tai 

Chuẩn đó anh,em khiếp khoản đó nhưng quen rồi  :icon6:




#623736 Topic về phương trình và hệ phương trình

Đã gửi bởi hoctrocuaHolmes on 30-03-2016 - 21:38 trong Phương trình - hệ phương trình - bất phương trình

Bài 353:Giải phương trình:$\sqrt[5]{x-1}+\sqrt[3]{x+8}=x^3+1$




#597455 Topic về Bất đẳng thức, cực trị THCS

Đã gửi bởi hoctrocuaHolmes on 08-11-2015 - 22:57 trong Bất đẳng thức và cực trị

cho x+y=1, CMR

x4+y4 $\geq$ $\frac{1}{8 }$

$x^{4}+y^{4}\geq \frac{(x^{2}+y^{2})^{2}}{2}\geq \frac{\frac{(x+y)^{2}}{4}}{2}=\frac{1}{8}(đpcm)$

Dấu ''='' xảy ra khi $x=y=\frac{1}{2}$




#583257 Topic về Bất đẳng thức, cực trị THCS

Đã gửi bởi hoctrocuaHolmes on 20-08-2015 - 09:35 trong Bất đẳng thức và cực trị

Cho $a,b,c>0$.CMR:

$\frac{a}{3a^2+2b^2+c^2}+\frac{b}{3b^2+2c^2+a^2}+\frac{c}{3c^2+2a^2+b^2}\leq \frac{1}{6}(\frac{1}{a}+\frac{1}{b}+\frac{1}{c})$

Ta có

$\frac{a}{3a^2+2b^2+c^2}=\frac{a}{2(a^{2}+b^{2})+(a^{2}+c^{2})}\leq \frac{a}{4ab+2ac}=\frac{1}{2}(\frac{1}{2b+c})=\frac{1}{2}(\frac{1}{b+b+c})\leq \frac{1}{18}(\frac{1}{b}+\frac{1}{b}+\frac{1}{c})=\frac{1}{18}(\frac{2}{b}+\frac{1}{c})$

Thiết lập các bất đẳng thức tương tự ta có 

$\frac{a}{3a^2+2b^2+c^2}+\frac{b}{3b^2+2c^2+a^2}+\frac{c}{3c^2+2a^2+b^2}\leq\frac{1}{18}(\frac{3}{a}+\frac{3}{b}+\frac{3}{c})=\frac{1}{6}(\frac{1}{a}+\frac{1}{b}+\frac{1}{c})(đpcm)$

Dấu''='' xảy ra khi $a=b=c$




#587184 $\boxed{{Topic}}$ Ôn thi học sinh giỏi lớp...

Đã gửi bởi hoctrocuaHolmes on 04-09-2015 - 15:10 trong Chuyên đề toán THCS

Ta có: $P=\frac{x^6}{(x^3+y^3)(x+y)} =\frac{x^6}{x^4+y^4 +xy(x^2+y^2)}$

 Dễ dàng chứng minh được $xy(x^2+y^2) \leq x^4+y^4$ nên $P=\frac{x^6}{(x^3+y^3)(x+y)} \geq \frac{x^6}{2(x^4+y^4)}$

$\Rightarrow 2A= \sum \frac{2x^6}{(x^3+y^3)(x+y)} \geq \sum \frac{x^6}{x^4+y^4}$

 Đặt $x^2 =a; y^2=b$ và $z^2=c$ ($a,b,c \geq 0$) thì $a+b+c=3$. Ta có:

$2A$$=$ $\sum \frac{a^3}{a^2+b^2} =\sum a -\sum \frac{ab^2}{a^2+b^2} \geq \sum a - \sum \frac{b}{2} =\frac{a+b+c}{2}$

$\Rightarrow 2A \geq \frac{3}{2} \Leftrightarrow A \geq \frac{3}{4}$

 Dấu $"="$ xảy ra khi $a=b=c=1 \Leftrightarrow x=y=z=1$

Áp dụng Cauchy-Schwarz và bất đẳng thức $x^2+y^2+z^2 \geq xy+yz+zx$ ta có

$VT=\sum \frac{(x^3+y^3)x^3-x^3y^3}{(x^3+y^3)(x+y)}=\sum \frac{x^3}{x+y}-\sum \frac{x^3y^3}{(x^3+y^3)(x+y)} \geq \sum \frac{x^4}{x^2+xy} -\sum \frac{x^3y^3}{xy(x+y)^2}\geq \frac{(x^2+y^2+z^2)^2}{x^2+y^2+z^2+xy+yz+zx}-\sum \frac{x^3y^3}{4x^2y^2}\geq \frac{(x^2+y^2+z^2)^2}{2(x^2+y^2+z^2)}-\frac{xy+yz+zx}{4} \geq \frac{3}{2}-\frac{x^2+y^2+z^2}{4} =\frac{3}{2}-\frac{3}{4}=\frac{3}{4}$

Dấu ''='' xảy ra khi $x=y=z=1$




#586964 $\boxed{{Topic}}$ Ôn thi học sinh giỏi lớp...

Đã gửi bởi hoctrocuaHolmes on 03-09-2015 - 11:01 trong Chuyên đề toán THCS

Ta có : $A^{2}=5+\sqrt{13+A}<=>(A-5)^{2}=13+A<=>(A-3)(A^{3}+3A^{2}-A-4)=0$

Em tự hỏi liệu biểu thức $A$ có bằng $3$ hay không bởi 

$A=\sqrt{5+\sqrt{13+\sqrt{5+\sqrt{13+......}}}}<\sqrt{5+\sqrt{13+\sqrt{5+\sqrt{13+3......}}}}=\sqrt{5+\sqrt{13+\sqrt{5+4}}}=3$

hay $A<3$.Do đó ta không thể tính giá trị của biểu thức $A$ ????

còn $A^3+3A^2-A-4=0$ lại cho ra nghiệm khá lẻ,vậy bài toán này giải như thế nào??




#587448 $\boxed{{Topic}}$ Ôn thi học sinh giỏi lớp...

Đã gửi bởi hoctrocuaHolmes on 05-09-2015 - 18:21 trong Chuyên đề toán THCS

Theo AM-GM ta có: $\frac{a^3}{b^2(c^2+d^2)}+\frac{b^2}{2}+\frac{c^2+d^2}{4}+\frac{a}{2}\geq$$ 2a^2$

 

Vậy nên: $\sum \frac{a^3}{b^2(c^2+d^2)}\geq \sum a^2-\frac{\sum a}{2}\geq \frac{t^2}{4}-\frac{t}{2}$

 

với $t=a+b+c+d$

 

Do đó cần chứng minh: $\frac{t^2}{4}-\frac{t}{2} \geq 2$

 

Tương đương với $(t-4)(t+2) \geq 0$ đúng theo AM-GM  $t=a+b+c+d \geq 4\sqrt[4]{abcd}=4$

Hình như đoạn màu đỏ có nhầm lẫn thì phải.Đó phải là $2a$ chứ bạn




#590457 $\boxed{{Topic}}$ Ôn thi học sinh giỏi lớp...

Đã gửi bởi hoctrocuaHolmes on 23-09-2015 - 15:22 trong Chuyên đề toán THCS

Cũng lâu rồi chưa đăng bài ở đây,mình xin đóng góp 1 số bài toán lấy từ 1 số nguồn(không có bài  hình)

Chúc TOPIC có nhiều bài đăng hay  :namtay

Bài 145:Rút gọn các biểu thức :

1) $P=\dfrac{m-n}{\sqrt m -\sqrt n}+m+n+2\sqrt{mn}$
với $m,n \geq 0, m \neq n$
2)$Q=\dfrac{a^2b-ab^2}{ab}:\dfrac{\sqrt a -\sqrt b}{\sqrt a+\sqrt b}$
với $a,b>0$

Bài 146:Chứng minh rằng các số có dạng $P= 2^{2m}+2^{2004}$ với m nguyên dương không thể là số chính phương

Bài 147:Tìm a, b để hệ sau có nghiệm duy nhất

\[
\left\{ \begin{array}{l}
xyz + z = a \\
xyz^2 + z = b \\
x^2 + y^2 + z^2 = 4 \\
\end{array} \right.
\]

Bài 148:Tìm nghiệm trong $[-1;1]$ của hệ phương trình ba ẩn sau:

$\left\{ \begin{array}{l}
x + y + z = 0 \\
{x^{2000}} + {y^{2002}} + {z^{2004}} = 2 \\
\end{array} \right.$

Bài 149:1) Cho $a, b, c$ là $3$ số không âm thỏa mãn điều kiện : $a^2+b^2+c^2 \leq 2(ab+bc+ca)$ (1) . Chứng minh bất đẳng thức :$(a+b+c) \leq 2( \sqrt{ab} + \sqrt{bc} + \sqrt{ca} ) $ (2)

Hỏi từ (2) có thể suy ra (1) hay không ? Vì sao ?
2) Cho a, b, c là 3 số không âm thỏa điều kiện (1) và p, q, r là các số thực thỏa $p+q+r=0$ . Chứng minh bất đẳng thức :

$ apq+bqr+crp \leq 0$




#586292 $\boxed{{Topic}}$ Ôn thi học sinh giỏi lớp...

Đã gửi bởi hoctrocuaHolmes on 30-08-2015 - 22:31 trong Chuyên đề toán THCS

76. Giải hệ phương trình:$\left\{\begin{matrix} x+y+z=5(1)\\ \frac{1}{x}+\frac{1}{y}+\frac{1}{z}=\frac{1}{5}(2)\\ y+2z^{2}=1(3) \end{matrix}\right.$

77. Tìm các số có 3 chữ số $\overline{abc}$ sao cho $\overline{abc}=9(a^{2}+b^{2}+c^{2})$

76. Từ pt $(1),(2)$ ta suy ra 

$\frac{1}{x}+\frac{1}{y}+\frac{1}{z}=\frac{1}{x+y+z}\Leftrightarrow (x+y)(y+z)(z+x)=0\Leftrightarrow \begin{bmatrix} x=-y\Rightarrow z=5;y=-49;x=49 & & \\ y=-z\Rightarrow x=5;\begin{bmatrix} y=-1;z=1 & \\ y=\frac{1}{2};z=\frac{-1}{2} & \end{bmatrix} & & \\ z=-x\Rightarrow y=5;2z^{2} =-4(VL)& & \end{bmatrix}$

Vậy bộ $(x,y,z)$ cần tìm là $(x,y,z)={(5;-1;1);(5;\frac{1}{2};\frac{-1}{2});(49;-49;5)}$




#598529 $\boxed{{Topic}}$ Ôn thi học sinh giỏi lớp...

Đã gửi bởi hoctrocuaHolmes on 15-11-2015 - 21:05 trong Chuyên đề toán THCS

 

Các anh, chị giải giúp em bộ đề này, em đang cần gấp ạ. Anh, chị nào giải được trọn bộ đề này trong vòng hôm nay cho tới trưa mai 16/11/15 em xin hậu tạ card 100k tự chọn mạng (phải nói trước) (trong vòng 7 ngày sẽ có)

 

Bài 1: CM đẳng thức: $\sqrt {\frac{{1 + \frac{{\sqrt 3 }}{2}}}{{1 + \sqrt {1 + \frac{{\sqrt 3 }}{2}} }} + \frac{{1 - \frac{{\sqrt 3 }}{2}}}{{1 - \sqrt {1 - \frac{{\sqrt 3 }}{2}} }}}  = 1$

Bài 2: a) Giải pt: $\sqrt {{x^2} - 1}  + \sqrt {10x - {x^2} - 9}  = \sqrt {2{x^2} - 14x + 12}$

b) Giải hpt $\left\{\begin{matrix} \sqrt {{x^2} + 2} + x + \sqrt {{y^2} + 3} + y = 5\\ \sqrt {{x^2} + 2} - x + \sqrt {{y^2} + 3} - y = 2 \end{matrix} \right.$

Bài 3: a) Trong mp tọa độ $Oxy$ cho đường thẳng $\left( d \right)$ có pt $\left( {m - 4} \right)x + \left( {m - 3} \right)y = 1$ (m là tham số). Tìm m để khoảng cách từ gốc tọa độ đến $\left( d \right)$ là nhỏ nhất / lớn nhất.

b) Tìm các số tự nhiên có hai chữ số $\overline {xy}$ sao cho $2\overline {xy}  = {\left( {x + 2} \right)^2} + {\left( {y + 1} \right)^2}$

Bài 4: Cho $a,b,c > 0$ thỏa $\sqrt {{a^2} + {b^2}}  + \sqrt {{b^2} + {c^2}}  + \sqrt {{c^2} + {a^2}}  = 1$

CMR: $\sqrt {\frac{{{a^2}}}{{b + c}} + \frac{{{b^2}}}{{c + a}} + \frac{{{c^2}}}{{a + b}}}  + \left( {a + b + c} \right) \ge \frac{{\sqrt[4]{2} + 2a + 2b + 2c}}{2}$

Em thật là hào phóng,chị thích em rồi đấy  :D

1. Ta có:

$\frac{{1 + \frac{{\sqrt 3 }}{2}}}{{1 + \sqrt {1 + \frac{{\sqrt 3 }}{2}} }} =\frac{2+\sqrt{3}}{2+2\sqrt{\frac{2+\sqrt{3}}{2}}}=\frac{2+\sqrt{3}}{2+\sqrt{4+2\sqrt{3}}}=\frac{2+\sqrt{3}}{2+\sqrt{3}+1}=\frac{2+\sqrt{3}}{3+\sqrt{3}}(1)$

$\frac{{1 - \frac{{\sqrt 3 }}{2}}}{{1 - \sqrt {1 - \frac{{\sqrt 3 }}{2}} }}= \frac{2-\sqrt{3}}{2-2\sqrt{\frac{2-\sqrt{3}}{2}}}= \frac{2-\sqrt{3}}{2-\sqrt{4-2\sqrt{3}}}=\frac{2-\sqrt{3}}{2-(\sqrt{3}-1)}=\frac{2-\sqrt{3}}{3-\sqrt{3}}(2)$

Từ $(1)(2)$ $\Rightarrow \sqrt {\frac{{1 + \frac{{\sqrt 3 }}{2}}}{{1 + \sqrt {1 + \frac{{\sqrt 3 }}{2}} }} + \frac{{1 - \frac{{\sqrt 3 }}{2}}}{{1 - \sqrt {1 - \frac{{\sqrt 3 }}{2}} }}} =\sqrt{\frac{2+\sqrt{3}}{3+\sqrt{3}}+\frac{2-\sqrt{3}}{3-\sqrt{3}}}=\sqrt{\frac{(2+\sqrt{3})(3-\sqrt{3})+(2-\sqrt{3})(3+\sqrt{3})}{6}}=\sqrt{\frac{6-2\sqrt{3}+3\sqrt{3}-3+6-3\sqrt{3}+2\sqrt{3}-3}{6}}=\sqrt{\frac{6}{6}}=1\rightarrow \blacksquare$

_____________________________________

2. a)ĐKXĐ:

$\left\{\begin{matrix} x^2\geq 1 & & \\ 10x - {x^2} - 9\geq 0 & & \\ x^{2}-7x+6\geq 0 & & \end{matrix}\right.$

Dễ thấy $x=1$ là một nghiệm của PT.

Xét $x$ khác $1$,chia cả 2 vế cho $\sqrt{x-1}$ ta có

$\sqrt{x+1}+\sqrt{9-x}=\sqrt{2(x-6)}\Leftrightarrow x+1+9-x+2\sqrt{10x - {x^2} - 9}=2x-12\Leftrightarrow 2\sqrt{10x - {x^2} - 9}=2x-22\Leftrightarrow \sqrt{10x - {x^2} - 9}=x-11(ĐK x\geq 11)\Leftrightarrow 10x - {x^2} - 9=(x-11)^{2}\Leftrightarrow x^{2}-16x+65=0\Leftrightarrow (x-8)^{2}+1>0\rightarrow ptvn$

Vậy pt có 1 nghiệm duy nhất là $x=1$

-----------------------------------------------------------

b)ĐK:Với mọi $x$ thuộc R

$\left\{\begin{matrix} \sqrt {{x^2} + 2} + x + \sqrt {{y^2} + 3} + y = 5(1)\\ \sqrt {{x^2} + 2} - x + \sqrt {{y^2} + 3} - y = 2 (2)\end{matrix} \right.$

Từ pt(2) ta có

$\sqrt {{x^2} + 2} - x + \sqrt {{y^2} + 3} - y = 2\Leftrightarrow \frac{2}{\sqrt {{x^2} + 2}+x}+\frac{3}{\sqrt {{y^2} + 3} +y }=2(3)$

Đặt $\sqrt {{x^2} + 2}+x=a;\sqrt {{y^2} + 3} +y=b(a,b\neq 0)\Rightarrow \left\{\begin{matrix} a+b=5 & \\ \frac{2}{a}+\frac{3}{b}=2 & \end{matrix}\right.\Leftrightarrow \left\{\begin{matrix} 2b+3a=2ab & \\ a+b=5 & \end{matrix}\right.\Leftrightarrow 2(5-a)+3a=2a(5-a)\Leftrightarrow 10+a=10a-2a^2\Leftrightarrow 2a^2-9a+10=0\Leftrightarrow (2a-5)(a-2)=0\Leftrightarrow \begin{bmatrix} a=\frac{5}{2} \Rightarrow b=\frac{5}{2}& \\ a=2\Rightarrow b=3 & \end{bmatrix}\Leftrightarrow \begin{bmatrix} \sqrt {{x^2} + 2}+x= \sqrt {{y^2} + 3} +y=\frac{5}{2}& \\ \left\{\begin{matrix} \sqrt {{x^2} + 2}+x=2 & \\ \sqrt {{y^2} + 3} +y=3 & \end{matrix}\right.& \end{bmatrix}$

Xét:

th1:$\sqrt {{x^2} + 2}+x= \sqrt {{y^2} + 3} +y=\frac{5}{2}\Leftrightarrow\left\{\begin{matrix} 2\sqrt{x^{2}+2}=5-2x( x\leq \frac{5}{2}) & \\ 2\sqrt{y^2+3}=5-2y(y\leq \frac{5}{2}) & \end{matrix}\right.\Leftrightarrow \left\{\begin{matrix} 4(x^{2}+2)=25+4x^{2}-20x & \\ 4(y^{2}+3)=25+4y^{2}-20y & \end{matrix}\right.\Leftrightarrow \left\{\begin{matrix} 20x=17 & \\ 20y=13 & \end{matrix}\right.\Leftrightarrow \left\{\begin{matrix} x=0,85 & \\ y=0,65 & \end{matrix}\right.(TM)$

th2:$\left\{\begin{matrix} \sqrt {{x^2} + 2}+x=2 & \\ \sqrt {{y^2} + 3} +y=3 & \end{matrix}\right.\Leftrightarrow \left\{\begin{matrix} \sqrt{x^{2}+2}=2-x(x\leq 2) & \\ \sqrt{y^2+3}=3-y(y\leq 3) & \end{matrix}\right.\Leftrightarrow \left\{\begin{matrix} {x^{2}+2=x^{2}-4x+4} & \\ y^2+3=y^{2}-6y+9 & \end{matrix}\right.\Leftrightarrow \left\{\begin{matrix} 4x=2 & \\ 6y=6 & \end{matrix}\right.\Leftrightarrow \left\{\begin{matrix} x=0,5 & \\ y=1& \end{matrix}\right.(TM)$

Vậy hệ có nghiệm là $(x,y)=(0,5;1);(0,85;0,65)$

_________________________________________

3.b)$2\overline {xy} = {( {x + 2} )^2} + {( {y + 1})^2}\Leftrightarrow 2(10x+y)=x^{2}+4x+4+y^2+2y+1\Leftrightarrow x^{2}+y^2-16x+5=0\Leftrightarrow (x-8)^2+y^2=59=1^{2}+58=2^{2}+55=3^2+50=4^2+43=5^2+34=6^2+23=7^2+10=0^2+59\rightarrow ptvn$

(do $x,y$ đều là các chữ số bé hơn $9$ và lớn hơn bằng $1$ nên $(x-8)^2$ và $y^2$ phải là các số chính phương)

Vậy không tồn tại chữ số $x,y$ nào thỏa mãn đề bài

_________________________________________

4. Bất đẳng thức cần cm tương đương vs

$\sqrt {\frac{{{a^2}}}{{b + c}} + \frac{{{b^2}}}{{c + a}} + \frac{{{c^2}}}{{a + b}}} + \left( {a + b + c} \right) \ge \frac{{\sqrt[4]{2} + 2a + 2b + 2c}}{2}\Leftrightarrow \sqrt {\frac{{{a^2}}}{{b + c}} + \frac{{{b^2}}}{{c + a}} + \frac{{{c^2}}}{{a + b}}} +(a+b+c)\geq \frac{\sqrt[4]{2}}{2}+(a+b+c)\Leftrightarrow \sqrt {\frac{{{a^2}}}{{b + c}} + \frac{{{b^2}}}{{c + a}} + \frac{{{c^2}}}{{a + b}}} \geq \frac{\sqrt[4]{2}}{2}\Leftrightarrow \frac{{{a^2}}}{{b + c}} + \frac{{{b^2}}}{{c + a}} + \frac{{{c^2}}}{{a + b}}\geq \frac{\sqrt{2}}{4}= \frac{1}{2\sqrt{2}}$

 Áp dụng AM-GM : $\sum \frac{a^2}{b+c}\geq \sum \frac{a^2}{\sqrt{2(b^2+c^2)}}$

 Đặt $\sqrt{b^2+c^2}=x;\sqrt{c^2+a^2}=y;\sqrt{a^2+b^2}=z$ thì $a^2=\frac{y^2+z^2-x^2}{2};b^2=\frac{x^2+z^2-y^2}{2};c^2=\frac{x^2+y^2-z^2}{2}$ và $x+y+z=1$

 $\Rightarrow \sum \frac{a^2}{b+c}\geq \frac{1}{2\sqrt{2}}.\sum \frac{y^2+z^2-x^2}{x^2}=\frac{1}{2\sqrt{2}}\left [ \sum \left (\frac{y^{2}+z^{2}}{x}+2x\right )-3.\sum x\right ]$

 $\geq \frac{1}{2\sqrt{2}}\left \{ \sum \left [\frac{(y+z)^2}{2x}+2x\right ]-3.\sum x \right \}$

 $\geq \frac{1}{2\sqrt{2}}.\left [2\sum (y+z)-3\sum x \right ]= \frac{1}{2\sqrt{2}}$ 

Dấu ''='' xảy ra khi $a=b=c=\frac{1}{3\sqrt{2}}$

_________________________________________

3.a) 

Với mọi $m$ đường thẳng $(d)$ không đi qua gốc tọa độ $(0;0)$

+$m=4$ ta có đường thẳng $y=1$ do đó khoảng cách từ $O$ đến $(d)$ là $1$ $(1)$

+$m=3$ ta có đường thẳng $x=-1$ do đó khoảng cách từ $O$ đến $(d)$ là $1$ $(2)$

+$m$ khác $4$ và $3$ thì $(d)$ cắt trục $Ox,Oy$ tại $A(0;\frac{1}{m-3});B(\frac{1}{m-4};0)$

Hạ $OH$ vuông góc với $AB$ trong tam giác vuông $AOB$ có

$OA=\frac{1}{\left | m-3 \right |};OB=\frac{1}{\left | m-4 \right |}\Rightarrow \frac{1}{OH^2}=\frac{1}{OA^2}+\frac{1}{OB^2}=(m-3)^2+(m-4)^2=2m^2-14m+25=2(m-\frac{7}{2})^2+\frac{1}{2}\geq \frac{1}{2}\Rightarrow OH^2\leq 2\Rightarrow OH\leq \sqrt{2}$

Dấu ''='' xảy ra khi $m=\frac{7}{2}$

P/s:Chị mạng Vinaphone nhé em trai  :D




#585147 $\boxed{{Topic}}$ Ôn thi học sinh giỏi lớp...

Đã gửi bởi hoctrocuaHolmes on 26-08-2015 - 20:37 trong Chuyên đề toán THCS

Bài 31:(Nguyên lí Dirichlet)

Có 1 nhóm 50 người đi xem phim buổi sáng.  Hàng ngang của rạp có 7 ghế, hàng dọc có 8 ghế. Chiều hôm đó cũng 50 người đó đi xem phim ở rạp này. Chứng minh rằng có ít nhất 2 người ngồi cùng 1  hàng ngang. 

Coi số thỏ là $50$,số chuồng là $7.8=56$ theo nguyên lí Dirichlet ta có $\left [ \frac{50}{56} \right ]+1=2$ 

Vậy có ít nhất $2$ người ngồi cùng một hàng ngang.




#584653 $\boxed{{Topic}}$ Ôn thi học sinh giỏi lớp...

Đã gửi bởi hoctrocuaHolmes on 24-08-2015 - 18:01 trong Chuyên đề toán THCS

b)Tìm các cặp số nguyên x,y biết $4y^{2}-\sqrt{199-x^{2}-2x}=2$

b)Ta có $4y^{2}-\sqrt{199-x^{2}-2x}=2\Leftrightarrow 4y^{2}=\sqrt{200-(x+1)^{2}}+2$

mà $0\leq \sqrt{200-(x+1)^{2}}\leq \sqrt{200}\Rightarrow 2\leq 4y^{2}\leq 2+10\sqrt{2}\Leftrightarrow \frac{1}{2}\leq y^{2}\leq \frac{5\sqrt{2}+1}{2}\Leftrightarrow y^{2}=1;y^{2}=4\Rightarrow y=\pm 1;y=\pm 2$

Xét $y=\pm 1\Rightarrow \begin{bmatrix} x=13 & \\ x=-15 & \end{bmatrix}$

$y=\pm 2\Rightarrow \begin{bmatrix} x=1 & \\ x=-3 & \end{bmatrix}$

Vậy...




#584642 $\boxed{{Topic}}$ Ôn thi học sinh giỏi lớp...

Đã gửi bởi hoctrocuaHolmes on 24-08-2015 - 17:25 trong Chuyên đề toán THCS

Bài 8: Khảo sát câu lạc bộ học sinh giỏi môn toán 9 quận Hoàn Kiếm năm học 2015-2016 có 90 học sinh tham gia. Kết quả khảo sát khi chấm thấy không có học sinh nào bị điểm dưới 9 và chỉ có một học sinh được 20 điểm. Biết rằng điểm bài khảo sát được làm tròn là số tự nhiên và thang điểm là 20 điểm. Chứng mình luôn tìm được ít nhất 8 học sinh có điểm khảo sát bằng nhau. 

( Trích Đề khảo sát câu lạc bộ Học sinh giỏi môn Toán lớp 9 năm học 2015-2016)

 

Ta có số học sinh được dưới 20 điểm là $90-1=89$(bạn)

Số điểm mà mỗi học sinh có thể nhận được là 9;10;11;12;13;14;15;16;17;18;19( vì số điểm là số tự nhiên)

Giả sử không tìm được ít nhất 8 học sinh nào có điểm khảo sát bằng nhau suy ra số học sinh phải nhỏ hơn $8.11=88$

mà lại có 89 học sinh nên mâu thuẫn suy ra đpcm

Bài này dùng Dirichlet sẽ nhanh hơn  :lol:

Coi số thỏ là $90$,số lồng là $12$ theo nguyên lí Dirichlet ta có $\left [ \frac{90}{12} \right ]+1=8$

Vậy có ít nhất $8$ học sinh điểm khảo sát bằng nhau




#584655 $\boxed{{Topic}}$ Ôn thi học sinh giỏi lớp...

Đã gửi bởi hoctrocuaHolmes on 24-08-2015 - 18:09 trong Chuyên đề toán THCS

c) Cho $B=\sqrt{6+\sqrt{6+\sqrt{6+...+\sqrt{6+\sqrt{6}}}}}+\sqrt[3]{6+\sqrt[3]{6+\sqrt[3]{6+...+\sqrt[3]{6+\sqrt[3]{6}}}}}$Mỗi vế có 2010 dấu căn

So sánh B với $\sqrt{26}$

Ta có $a_{1}=\sqrt{6}< 3\Rightarrow \sqrt{6+a_{1}}< 3\Rightarrow ...\Rightarrow \sqrt{6+\sqrt{6+\sqrt{6+...+\sqrt{6+\sqrt{6}}}}}< 3$

$(1)$

$b_{1}=\sqrt[3]{6}< 2\Rightarrow \sqrt[3]{6+b_{1}}< 2\Rightarrow ...\Rightarrow \sqrt[3]{6+\sqrt[3]{6+\sqrt[3]{6+...+\sqrt[3]{6+\sqrt[3]{6}}}}}< 2 (2)$

Từ $(1)(2)$ suy ra

$B< 2+3=5=\sqrt{25}< \sqrt{26}$




#585158 $\boxed{{Topic}}$ Ôn thi học sinh giỏi lớp...

Đã gửi bởi hoctrocuaHolmes on 26-08-2015 - 20:48 trong Chuyên đề toán THCS

Bài 32:(Bất biến)

Cho 1 hình vuông 4x4 có 1 ô đen 15 ô trắng. Chúng ta có thể đổi màu tất cả các ô hàng ngang hay hàng dọc từ trắng sang đen hay từ đen sang trắng. Hãy cho biết sau nhiều lần đổi thì các ô có cùng màu không

Quy ước ô đen là số $1$ và ô trắng là số $-1$ để cả $16$ ô cùng màu thì tích của chúng phải là một số dương

Vì ta đổi màu tất cả các ô hàng ngang hay hàng dọc từ trắng sang đen hay từ đen sang trắng nên tích các số vẫn không thay đổi và luôn nhận giá trị là $-1$ (mâu thuẫn).Do đó sau nhiều lần đổi thì các ô vẫn không cùng màu.Đây chính là bất biến của bài toán.




#585939 $\boxed{{Topic}}$ Ôn thi học sinh giỏi lớp...

Đã gửi bởi hoctrocuaHolmes on 30-08-2015 - 08:08 trong Chuyên đề toán THCS

Bài 30:

​Cho $a_{1};a_{2};a_{3};a_{4};a_{5}\in Z$

C/m rằng $P=(a_{1}-a_{2})(a_{1}-a_{3})(a_{1}-a_{4})(a_{1}-a_{5})(a_{2}-a_{3})(a_{2}-a_{4})(a_{2}-a_{5})(a_{3}-a_{4})(a_{3}-a_{5})(a_{4}-a_{5})\vdots 144$

Xét:th1:$2$ trong $5$ số trên bằng nhau thì hiển nhiên $P=0$ chia hết cho $144$

th2:$a_{1};a_{2};a_{3};a_{4};a_{5}$ phân biệt

Theo nguyên lí Dirichlet trong $4$ số $a_{1};a_{2};a_{3};a_{4}$ tồn tại $2$ số có cùng số dư khi chia cho $3$ nên hiệu của chúng chia hết cho $3$.Không mất tính tổng quát giả sử $(a_{1}-a_{2})\vdots 3$

Tương tự xét $4$ số $a_{2};a_{3};a_{4};a_{5}$ ta lại được $2$ số khác có hiệu chia hết cho $3$

Do đó $P\vdots 9$ $(1)$

Cũng theo Dirichlet tồn tại $3$ số có cùng tính chẵn lẻ trong $5$ số đã cho.Xét:

th1:Nếu có ít nhất $4$ số có cùng tính chẵn lẻ thì ta sẽ có 6 hiệu của chúng chia hết cho $2$.Do đó tích của chúng chia hết cho $32$ hay $P$ chia hết cho $32$ $(2)$

th2:Nếu có đúng $3$ số có cùng tính chẵn lẻ.Không mất tính tổng quát giả sử $3$ số đó là $a_{1};a_{2};a_{3}$,Khi đó $2$ số còn lại $a_{4};a_{5}$ cùng tính chẵn lẻ giống nhau nhưng khác vs $3$ số kia.Do đó có $4$ hiệu chia hết cho $2$ hay $P$ chia hết cho $16$ $(3)$

Từ $(1)(2)(3)$ suy ra $P$ chia hết cho $16.9=144$ (vì $(2,3=1)$)

Spoiler




#585609 $\boxed{{Topic}}$ Ôn thi học sinh giỏi lớp...

Đã gửi bởi hoctrocuaHolmes on 28-08-2015 - 19:45 trong Chuyên đề toán THCS

$\boxed{ Bài 44}$Cho đa thức $P( x  )=x^{2}+bx+c$, trong đó b và c là các số nguyên. Biết rằng đa thức $x^{4}+6x+25$ và $3x^{4}+4x^{2}+28x+5$ đều chia hết cho $P( x)$. Tính $P( 1)$

$\boxed{ Bài 45}$Giải phương trình: $\frac{( b-c)( 1+a)^{2}}{x+a^{2}}+\frac{( c-a )( 1+b)^{2}}{x+b^{2}}+\frac{ ( a-b)( 1+c)^{2}}{x+c^{2}}=0$ ($a, b, c$ là các hằng số và đôi một khác nhau)

$\boxed{ Bài 46}$ Giải phương trình với $x$ là số thực dương $\sqrt[3]{4x^{2}+6x+1}+\sqrt[3]{2x^{2}+3x+1}=2x^{2}+3x+2$




#585942 $\boxed{{Topic}}$ Ôn thi học sinh giỏi lớp...

Đã gửi bởi hoctrocuaHolmes on 30-08-2015 - 08:25 trong Chuyên đề toán THCS

Bài 63:Cho $a,b,c\geq 1$ . Chứng minh rằng

$\sqrt{a-1}+\sqrt{b-1}+\sqrt{c-1}\leq \sqrt{c(ab-1)}$

Đoạn màu đỏ phải là dấu ''+'' chứ nếu không thay $a=b=1;c=2$ thấy ngay bất đẳng thức sai  :mellow:

Áp dụng Bunhiacopxki ta có

$(\sqrt{a-1}+\sqrt{b-1}+\sqrt{c-1})^{2}\leq (c-1+1)[(\sqrt{a-1}+\sqrt{b-1})^{2}+1]=c[(\sqrt{a+1}+\sqrt{b-1})^{2}+1]\Leftrightarrow \sqrt{a-1}+\sqrt{b-1}+\sqrt{c-1}\leq \sqrt{c[(\sqrt{a+1}+\sqrt{b-1})^{2}+1]}$

Ta cần cm $\sqrt{c[(\sqrt{a-1}+\sqrt{b-1})^{2}+1]}\leq \sqrt{c(ab+1)}\Leftrightarrow (\sqrt{a-1}+\sqrt{b-1})^{2}+1\leq (ab+1)\Leftrightarrow 2\sqrt{(a-1)(b-1)}+a+b-1\leq ab+1\Leftrightarrow2\sqrt{(a-1)(b-1)}\leq ab-a-b +2=(a-1)(b-1)+1\Leftrightarrow (\sqrt{(a-1)(b-1)}-1)^{2}\geq 0$ 

(luôn đúng)

Dấu ''='' xảy ra khi $a=b=2$;$c=\frac{5}{4}$